ΔABC is similar to ΔMNO. The scale factor from ΔMNO to ΔABC is 3∕2 . If the area of ΔMNO is 10 square units, what's the area of ΔABC? Question 12 options: A) 45 square units B) 90 square units C) 22.5 square units D) 15 square units

Answers

Answer 1

Answer:

The area of ΔABC= 6.667 square units

Step-by-step explanation:

ΔABC is similar to ΔMNO.

The scale factor from ΔMNO to ΔABC is 3∕2

the area of ΔMNO is 10 square units,

The area of ΔABC/the area of ΔMNO

= 2/3

The area of ΔABC/10= 2/3

The area of ΔABC= 2/3 * 10

The area of ΔABC= 20/3

The area of ΔABC= 6 2/3

The area of ΔABC= 6.667 square units

Answer 2

Answer:

22.5 square units

Step-by-step explanation:

i multiplied 10 by 2 to get 20 and went with the closest answer and got it right.

i dont know how to do math but i guess it worked


Related Questions

HELP ASAP PLS :Find all the missing elements:

Answers

Answer:

a ≈ 1.59

b ≈ 6.69

Step-by-step explanation:

Law of Sines: [tex]\frac{a}{sinA} =\frac{b}{sinB} =\frac{c}{sinC}[/tex]

Step 1: Find c using Law of Sines

[tex]\frac{6}{sin58} =\frac{c}{sin13}[/tex]

[tex]c = sin13(\frac{6}{sin58})[/tex]

c = 1.59154

Step 2: Find a using Law of Sines

[tex]\frac{6}{sin58} =\frac{a}{sin109}[/tex]

[tex]a = sin109(\frac{6}{sin58} )[/tex]

a = 6.68961

The area of a rectangular garden if 6045 ft2. If the length of the garden is 93 feet, what is its width?

Answers

Answer:

65 ft

Step-by-step explanation:

The area of a rectangle is

A = lw

6045 = 93*w

Divide each side by 93

6045/93 = 93w/93

65 =w

Answer:

[tex]\huge \boxed{\mathrm{65 \ feet}}[/tex]

Step-by-step explanation:

The area of a rectangle formula is given as,

[tex]\mathrm{area = length \times width}[/tex]

The area and length are given.

[tex]6045=93 \times w[/tex]

Solve for w.

Divide both sides by 93.

[tex]65=w[/tex]

The width of the rectangular garden is 65 feet.

Express the product of z1 and z2 in standard form given that [tex]z_{1} = 6[cos(\frac{2\pi }{5}) + isin(\frac{2\pi }{5})][/tex] and [tex]z_{2} = 2\sqrt{2} [cos(\frac{-\pi }{2}) + isin(\frac{-\pi }{2})][/tex]

Answers

Answer:

Solution : 5.244 - 16.140i

Step-by-step explanation:

If we want to express the two as a product, we would have the following expression.

[tex]-6\left[\cos \left(\frac{2\pi }{5}\right)+i\sin \left(\frac{2\pi }{5}\right)\right]\cdot 2\sqrt{2}\left[\cos \left(\frac{-\pi }{2}\right)+i\sin \left(\frac{-\pi \:}{2}\right)\right][/tex]

Now we have two trivial identities that we can apply here,

( 1 ) cos(- π / 2) = 0,

( 2 ) sin(- π / 2) = - 1

Substituting them,

= [tex]-6\cdot \:2\sqrt{2}\left(0-i\right)\left(\cos \left(\frac{2\pi }{5}\right)+i\sin \left(\frac{2\pi }{5}\right)\right)[/tex]

= [tex]-12\sqrt{2}\sin \left(\frac{2\pi }{5}\right)+12\sqrt{2}\cos \left(\frac{2\pi }{5}\right)i[/tex]

Again we have another two identities we can apply,

( 1 ) sin(x) = cos(π / 2 - x )

( 2 ) cos(x) = sin(π / 2 - x )

[tex]\sin \left(\frac{2\pi }{5}\right)=\cos \left(\frac{\pi }{2}-\frac{2\pi }{5}\right) = \frac{\sqrt{2}\sqrt{5+\sqrt{5}}}{4}[/tex]

[tex]\cos \left(\frac{2\pi }{5}\right)=\sin \left(\frac{\pi }{2}-\frac{2\pi }{5}\right) = \frac{\sqrt{2}\sqrt{3-\sqrt{5}}}{4}[/tex]

Substitute,

[tex]-12\sqrt{2}(\frac{\sqrt{2}\sqrt{5+\sqrt{5}}}{4}) + 12\sqrt{2}(\frac{\sqrt{2}\sqrt{3-\sqrt{5}}}{4})[/tex]

= [tex]-6\sqrt{5+\sqrt{5}}+6\sqrt{3-\sqrt{5}} i[/tex]

= [tex]-16.13996 + 5.24419i[/tex]

= [tex]5.24419i - 16.13996[/tex]

As you can see option d is the correct answer. 5.24419 is rounded to 5.244, and 16.13996 is rounded to 16.14.

solve the system with elimination 4x+3y=1 -3x-6y=3

Answers

Answer:

x = 1, y = -1

Step-by-step explanation:

If we have the two equations:

[tex]4x+3y=1[/tex] and [tex]-3x - 6y = 3[/tex], we can look at which variable will be easiest to eliminate.

[tex]y[/tex] looks like it might be easy to get rid of, we just have to multiply [tex]4x+3y=1[/tex]  by 2 and y is gone (as -6y + 6y = 0).

So let's multiply the equation [tex]4x+3y=1[/tex]  by 2.

[tex]2(4x + 3y = 1)\\8x + 6y = 2[/tex]

Now we can add these equations

[tex]8x + 6y = 2\\-3x-6y=3\\[/tex]

------------------------

[tex]5x = 5[/tex]

Dividing both sides by 5, we get [tex]x = 1[/tex].

Now we can substitute x into an equation to find y.

[tex]4(1) + 3y = 1\\4 + 3y = 1\\3y = -3\\y = -1[/tex]

Hope this helped!

how would you write six times the square of a number

Answers

6 to the power of whatever number you are going by

Answer:

[tex]\huge \boxed{6x^2 }[/tex]

Step-by-step explanation:

6 times a number squared.

Let the number be [tex]x[/tex].

6 is multiplied to [tex]x[/tex] squared.

[tex]6 \times x^2[/tex]

There are 30 colored marbles inside a bag. Six marbles are yellow, 9 are red, 7 are white, and 8 are blue. One is drawn at random. Which color is most likely to be chosen? A. white B. red C. blue D. yellow Include ALL work please!

Answers

Answer:

red

Step-by-step explanation:

Since the bag contains more red marbles than any other color, you are most likely to pick a red marble


An apartment building is infested with 6.2 X 10 ratsOn average, each of these rats
produces 5.5 X 10' offspring each year. Assuming no rats leave or die, how many additional
rats will live in this building one year from now? Write your answer in standard form.

Answers

Answer: 3.41x10^3

Step-by-step explanation:

At the beginning of the year, we have:

R = 6.2x10 rats.

And we know that, in one year, each rat produces:

O = 5.5x10 offsprins.

Then each one of the 6.2x10 initial rats will produce 5.5x10 offsprings in one year, then after one year we have a total of:

(6.2x10)*(5.5x10) = (6.2*5.5)x(10*10) = 34.1x10^2

and we can write:

34.1 = 3.41x10

then: 34.1x10^2 = 3.41x10^3

So after one year, the average number of rats is:  3.41x10^3

Suppose that $2000 is invested at a rate of 2.6% , compounded semiannually. Assuming that no withdrawals are made, find the total amount after 10 years.

Answers

Answer:

$2,589.52

Step-by-step explanation:

[tex] A = P(1 + \dfrac{r}{n})^{nt} [/tex]

We start with the compound interest formula above, where

A = future value

P = principal amount invested

r = annual rate of interest written as a decimal

n = number of times interest is compound per year

t = number of years

For this problem, we have

P = 2000

r = 0.026

n = 2

t = 10,

and we find A.

[tex] A = $2000(1 + \dfrac{0.026}{2})^{2 \times 10} [/tex]

[tex] A = $2589.52 [/tex]

Compound interest formula:

Total = principal x ( 1 + interest rate/compound) ^ (compounds x years)

Total = 2000 x 1+ 0.026/2^20

Total = $2,589.52


8.What side of the road will you see speed, yield, and guide signs on ?

Answers

Answer:

we see it in our left side of the road

you will be able to spot these signs on your left hand side :)

A restaurant hands out a scratch-off game ticket with prizes being worth purchases at the restaurant. The back of the ticket lists the odds of winning each dollar value: 0.05 for $10, 0.04 for $25, 0.01 for $50, and 0.003 for $100. What are the odds that the ticket is worth at least $25?

Answers

Answer: 0.05412

Step-by-step explanation:

Formula : Odds of having an event is given by  [tex]o=\dfrac{p}{1-p}[/tex], where p = probability that event happens.

In terms to find p , we use [tex]p=\dfrac{o}{1+o}[/tex]

Given, he back of the ticket lists the odds of winning each dollar value: 0.05 for $10, 0.04 for $25, 0.01 for $50, and 0.003 for $100.

Let X be the worth of ticket.

Then, the probability that the ticket is worth at least $25 =

[tex]P(X\geq 25)=P(X=25)+P(X=50)+P(X=100)[/tex]

[tex]=\dfrac{0.04}{1+0.04}+\dfrac{0.01}{1+0.01}+\dfrac{0.003}{1+0.003}\\\\=0.05135[/tex]

The odds that the ticket is worth at least $25 = [tex]\dfrac{0.05135}{1-0.05135}[/tex]

=0.05412

hence, the odds that the ticket is worth at least $25 is 0.05412 .

Let E and F be two events of an experiment with sample space S. Suppose P(E) = 0.6, P(F) = 0.3, and P(E ∩ F) = 0.1. Compute the values below.

(a) P(E ∪ F) =



(b) P(Ec) =



(c) P(Fc ) =



(d) P(Ec ∩ F) =

Answers

Answer:

(a) P(E∪F)= 0.8

(b) P(Ec)= 0.4

(c) P(Fc)= 0.7

(d) P(Ec∩F)= 0.8

Step-by-step explanation:

(a) It is called a union of two events A and B, and A ∪ B (read as "A union B") is designated to the event formed by all the elements of A and all of B. The event A∪B occurs when they do A or B or both.

If the events are not mutually exclusive, the union of A and B is the sum of the probabilities of the events together, from which the probability of the intersection of the events will be subtracted:

P(A∪B) = P(A) + P(B) - P(A∩B)

In this case:

P(E∪F)= P(E) + P(F) - P(E∩F)

Being P(E) = 0.6, P(F) = 0.3 and P(E ∩ F) = 0.1

P(E∪F)= 0.6 + 0.3 - 0.1

P(E∪F)= 0.8

(b)  The complement of an event A is defined as the set that contains all the elements of the sample space that do not belong to A.  The Complementary Rule establishes that the sum of the probabilities of an event and its complement must be equal to 1. So, if P (A) is the probability that an event A occurs, then the probability that A does NOT occur is  P (Ac) = 1- P (A)

In this case: P(Ec)= 1 - P(E)

Then: P(Ec)= 1 - 0.6

P(Ec)= 0.4

(c) In this case: P(Fc)= 1 - P(F)

Then: P(Fc)= 1 - 0.3

P(Fc)= 0.7

(d)  The intersection of two events A and B, designated as A ∩ B (read as "A intersection B") is the event formed by the elements that belong simultaneously to A and B. The event A ∩ B occurs when A and B do at once.

As mentioned, the complementary rule states that the sum of the probabilities of an event and its complement must equal 1. Then:

P(Ec intersection F) + P(E intersection F) = P(F)

P(Ec intersection F) + 0.1 = 0.3

P(Ec intersection F)= 0.2

Being:

P(Ec∪F)= P(Ec) + P(F) - P(Ec∩F)

you get:

P(Ec∩F)= P(Ec) + P(F) - P(Ec∪F)

So:

P(Ec∩F)= 0.4 + 0.3 - 0.2

P(Ec∩F)= 0.8

Which choice shows the product of 22 and 49 ?

Answers

Answer:

1078

Step-by-step explanation:

The product of 22 and 49 is 1078.

Answer:

1078 is the product

Step-by-step explanation:

Identifying the Property of Equality

Quick

Check

Identify the correct property of equality to solve each equation.

3+x= 27

X/6 = 5

Answers

Answer:

a) Compatibility of Equality with Addition, b) Compatibility of Equality with Multiplication

Step-by-step explanation:

a) This expression can be solved by using the Compatibility of Equality with Addition, that is:

1) [tex]3+x = 27[/tex] Given

2) [tex]x+3 = 27[/tex] Commutative property

3) [tex](x + 3)+(-3) = 27 +(-3)[/tex] Compatibility of Equality with Addition

4) [tex]x + [3+(-3)] = 27+(-3)[/tex] Associative property

5) [tex]x + 0 = 27-3[/tex] Existence of Additive Inverse/Definition of subtraction

6) [tex]x=24[/tex] Modulative property/Subtraction/Result.

b) This expression can be solved by using the Compatibility of Equality with Multiplication, that is:

1) [tex]\frac{x}{6} = 5[/tex] Given

2) [tex](6)^{-1}\cdot x = 5[/tex] Definition of division

3) [tex]6\cdot [(6)^{-1}\cdot x] = 5 \cdot 6[/tex] Compatibility of Equality with Multiplication

4) [tex][6\cdot (6)^{-1}]\cdot x = 30[/tex] Associative property

5) [tex]1\cdot x = 30[/tex] Existence of multiplicative inverse

6) [tex]x = 30[/tex] Modulative property/Result

Answer:

3 + x = 27

✔ subtraction property of equality with 3

x over 6  = 5

✔ multiplication property of equality with 6

can anyone show me this in verbal form?

Answers

Answer:

2 * (x + 2) = 50

Step-by-step explanation:

Let's call the unknown number x. "A number and 2" means that we need to add the numbers, therefore it would be x + 2. "Twice" means 2 times a quantity so "twice a number and 2" would be 2 * (x + 2). "Is" denotes that we need to use the "=" sign and because 50 comes after "is", we know that 50 goes on the right side of the "=" so the final answer is 2 * (x + 2) = 50.

What are the solutions of the equation x4 + 6x2 + 5 = 0? Use u substitution to solve.
x = i and x = i5
x=+ i and x
x= +115
O x=V-1 and x = = -5
x=+ -1 and x = = -5​

Answers

Answer:

A; The first choice.

Step-by-step explanation:

We have the equation [tex]x^4+6x^2+5=0[/tex] and we want to solve using u-substitution.

When solving by u-substitution, we essentially want to turn our equation into quadratic form.

So, let [tex]u=x^2[/tex]. We can rewrite our equation as:

[tex](x^2)^2+6(x^2)+5=0[/tex]

Substitute:

[tex]u^2+6u+5=0[/tex]

Solve. We can factor:

[tex](u+5)(u+1)=0[/tex]

Zero Product Property:

[tex]u+5=0\text{ and } u+1=0[/tex]

Solve for each case:

[tex]u=-5\text{ and } u=-1[/tex]

Substitute back u:

[tex]x^2=-5\text{ and } x^2=-1[/tex]

Take the square root of both sides for each case. Since we are taking an even root, we need plus-minus. Thus:

[tex]x=\pm\sqrt{-5}\text{ and } x=\pm\sqrt{-1}[/tex]

Simplify:

[tex]x=\pm i\sqrt{5}\text{ and } x=\pm i[/tex]

Our answer is A.

Find the value of x show your work

Answers

Answer:

x≈13.08

Step-by-step explanation:

We use the pythagora's theorem

[tex]a^{2} +b^2=c^2\\a=5\\b=x\\c=14\\5^2+x^2=14^2\\x^2=196-25\\x^2=171\\x=3\sqrt{19} =13.08[/tex]

A thin metal plate, located in the xy-plane, has temperature T(x, y) at the point (x, y). Sketch some level curves (isothermals) if the temperature function is given by

T(x, y)= 100/1+x^2+2y^2

Answers

Answer:

Step-by-step explanation:

Given that:

[tex]T(x,y) = \dfrac{100}{1+x^2+y^2}[/tex]

This implies that the level curves of a function(f) of two variables relates with the curves with equation f(x,y) = c

here c is the constant.

[tex]c = \dfrac{100}{1+x^2+2y^2} \ \ \--- (1)[/tex]

By cross multiply

[tex]c({1+x^2+2y^2}) = 100[/tex]

[tex]1+x^2+2y^2 = \dfrac{100}{c}[/tex]

[tex]x^2+2y^2 = \dfrac{100}{c} - 1 \ \ -- (2)[/tex]

From (2); let assume that the values of c > 0 likewise c < 100, then the interval can be expressed as 0 < c <100.

Now,

[tex]\dfrac{(x)^2}{\dfrac{100}{c}-1 } + \dfrac{(y)^2}{\dfrac{50}{c}-\dfrac{1}{2} }=1[/tex]

This is the equation for the  family of the eclipses centred at (0,0) is :

[tex]\dfrac{x^2}{a^2}+\dfrac{y^2}{b^2}=1[/tex]

[tex]a^2 = \dfrac{100}{c} -1 \ \ and \ \ b^2 = \dfrac{50}{c}- \dfrac{1}{2}[/tex]

Therefore; the level of the curves are all the eclipses with the major axis:

[tex]a = \sqrt{\dfrac{100 }{c}-1}[/tex]  and a minor axis [tex]b = \sqrt{\dfrac{50 }{c}-\dfrac{1}{2}}[/tex]  which satisfies the values for which 0< c < 100.

The sketch of the level curves can be see in the attached image below.

Hi i need help on this im not that smart sorry, what is the x-intercept of the graph that is shown below

Answers

Answer:

(3, 0)

Step-by-step explanation:

x-intercept is where the line touches the x-axis

It is the point on the line where y=0

Answer:

3,0

Step-by-step explanation:

the point where the line cuts the x axis is the x-intecept

Given a right triangle with a hypotenuse of 6 cm and a leg of 4cm, what is the measure of the other leg of the triangle rounded to the tenths?

Answers

Answer:

4.5 cm

Step-by-step explanation:

a^2+b^2=c^2

A represents the leg we already know, which has a length of 4 cm. C represents the hypotenuse with a length of 6 cm:

4^2+b^2=6^2, simplified: 16+b^2=36

subtract 16 from both sides:

b^2=20

now find the square root of both sides and that is the length of the other leg.

sqrt20= 4.4721, which can be rounded to 4.5

Answer:

4.5 cm

Step-by-step explanation:

Since this is a right triangle, we can use the Pythagorean Theorem.

[tex]a^2+b^2=c^2[/tex]

where a and b are the legs and c is the hypotenuse.

One leg is unknown and the other is 4 cm. The hypotenuse is 6 cm.

[tex]a=a\\b=4\\c=6[/tex]

Substitute the values into the theorem.

[tex]a^2+4^2=6^2[/tex]

Evaluate the exponents first.

4^2= 4*4= 16

[tex]a^2+16=6^2[/tex]

6^2=6*6=36

[tex]a^2+16=36[/tex]

We want to find a, therefore we must get a by itself.

16 is being added on to a^2. The inverse of addition is subtraction. Subtract 16 from both sides of the equation.

[tex]a^2+16-16=36-16\\\\a^2=36-16\\\\a^2=20[/tex]

a is being squared. The inverse of a square is a square root. Take the square root of both sides.

[tex]\sqrt{a^2}=\sqrt{20} \\\\a=\sqrt{20} \\\\a=4.47213595[/tex]

Round to the nearest tenth. The 7 in the hundredth place tells us to round the 4 in the tenth place to a 5.

[tex]a=4.5[/tex]

Add appropriate units. In this case, centimeters.

a= 4.5 cm

The length of the other leg is about 4.5 centimeters.

I need help on this question :(​

Answers

Answer: 40 degree

Explanation:

FT bisect angle EFD dividing it into 2 equal angles (EFT and DFT)

And EFD = 80

We get :
EFT = 80/2
EFT = 40

And EFT + DFT = EFD = 80 degree

Therefore EFT = 40 degrees

The general manager, marketing director, and 3 other employees of CompanyAare hosting a visitby the vice president and 2 other employees of CompanyB. The eight people line up in a randomorder to take a photo. Every way of lining up the people is equally likely.Required:a. What is the probability that the bride is next to the groom?b. What is the probability that the maid of honor is in the leftmost position?c. Determine whether the two events are independent. Prove your answer by showing that one of the conditions for independence is either true or false.

Answers

Answer:

Following are the answer to this question:

Step-by-step explanation:

Let, In the Bth place there are 8 values.

In point a:

There is no case, where it generally manages its next groom is = 7 and it will be arranged in the 2, that can be arranged in 2! ways. So, the total number of ways are: [tex]\to 7 \times 2= 14\\\\ \{(1,2),(2,1),(2,3),(3,2),(3,4),(4,3),(4,5),(5,4),(5,6),(6,5),(6,7),(7,8),(8,7),(7,6)\}\\[/tex][tex]\therefore[/tex] required probability:

[tex]= \frac{14}{8!}\\\\= \frac{14}{8\times7 \times6 \times 5 \times 4 \times 3\times 2 \times 1 }\\\\= \frac{1}{8\times6 \times5 \times 4 \times 3}\\\\= \frac{1}{8\times6 \times5 \times 4 \times 3}\\\\=\frac{1}{2880}\\\\=0.00034[/tex]

In point b:

Calculating the leftmost position:

[tex]\to \frac{7!}{8!}\\\\\to \frac{7!}{8 \times 7!}\\\\\to \frac{1}{8}\\\\\to 0.125[/tex]

In point c:

This option is false because

[tex]\to P(A \cap B) \neq P(A) \times P(B)\\\\\to \frac{12}{8!} \neq \frac{14}{8!}\times \frac{1}{8}\\\\\to \frac{12}{8!} \neq \frac{7}{8!}\times \frac{1}{4}\\\\[/tex]

PLS HELPPPPPPPPPPP :p 8*10^3 is how many times larger that 4*10^2?

Answers

Answer:

20 times.

Step-by-step explanation:

To find out how many times larger a number is than another number, simply divide the two numbers, with the larger number being in the numerator.

For example, how many times larger is 6 than 2? The answer would be 6/2 or 3 times larger.

So, divide 8*(10^3) and 4*(10^2):

[tex]\frac{8\times10^3}{4\times10^2}[/tex]

Expand the expressions. This is the same as saying:

[tex]\frac{8\times10\times10\times10}{4\times10\times10}[/tex]

We can cancel two of the 10s since they are in both the numerator and the denominator. Thus, only one 10 is left in the numerator:

[tex]\frac{8\times10}{4}[/tex]

Simplify:

[tex]=\frac{80}{4} =20[/tex]

Therefore, 8*(10^3) (or 8000) is 20 times larger than 4*(10^2) (or 400).

Answer:

20 times

Step-by-step explanation:

hey,

so lets solve 8*10^3  first

so we use the order of operations

P

= Parentheses first

E

= Exponents (ie Powers and Square Roots, etc.)

MD

= Multiplication and Division (left-to-right)

AS

= Addition and Subtraction (left-to-right)

so  after doing the exponents part 8*1000

we do the multiplication

=8000

SO THE FIRST NUMBER IS 8000

now lets solve 4*10^2

so we use the order of operations

P

= Parentheses first

E

= Exponents (ie Powers and Square Roots, etc.)

MD

= Multiplication and Division (left-to-right)

AS

= Addition and Subtraction (left-to-right)

so we do exponents first 4*100

then multiplication

=400

SO THE SECOND NUMBER IS 400

To find out how many times larger a number is than another number, simply divide the two numbers, with the larger number being in the numerator.

now we divide  8000 by 400

=20

so 8*10^3 is 20 times larger than  4*10^2

HOPE I HELPED

PLS MARK BRAINLIEST  

DESPERATELY TRYING TO LEVEL UP

✌ -ZYLYNN JADE ARDENNE

JUST A RANDOM GIRL WANTING TO HELP PEOPLE!

                    PEACE!

given point (-6, -3) and a slope of 4, write an equation in point-slope form

Answers

Answer:

y = 4x + 21

Step-by-step explanation:

Hello!

Point-slope form is y - y1 = m(x - x1)

y1 is the y point

x1 is the x point

m is the slope

Put in what you know

y - -3 = 4(x - -6)

Subtracting a negative is the same as adding

y + 3 = 4(x + 6)

Distribute the 4

y + 3 = 4x + 24

Subtract 3 from both sides

y = 4x + 21

The answer is y = 4x + 21

Hope this helps!

Simplify to create an equivalent expression.
\qquad{7n-(4n-3)}7n−(4n−3)

Answers

Answer:

[tex]3n + 3[/tex]

[tex]3(n+1)[/tex]

Step-by-step explanation:

Given

[tex]7n - (4n - 3)[/tex]

Required

Simplify

To simplify the given expression, you start by opening  the bracket

[tex]7n - (4n - 3)[/tex]

[tex]7n - 4n + 3[/tex]

Next, you perform arithmetic operations on like terms

[tex]3n + 3[/tex]

The answer can be further simplified;

Factorize [tex]3n + 3[/tex]

[tex]3(n+1)[/tex]

Hence;

[tex]7n - (4n - 3)[/tex] when simplified is equivalent to [tex]3n + 3[/tex] or [tex]3(n+1)[/tex]

Answer:

3n+n

Step-by-step explanation:

Please give me the answer ASAP The average of 5 numbers is 7. If one of the five numbers is removed, the average of the four remaining numbers is 6. What is the value of the number that was removed Show Your Work

Answers

Answer:

The removed number is 11.

Step-by-step explanation:

Given that the average of 5 numbers is 7. So you have to find the total values of 5 numbers :

[tex]let \: x = total \: values[/tex]

[tex] \frac{x}{5} = 7[/tex]

[tex]x = 7 \times 5[/tex]

[tex]x = 35[/tex]

Assuming that the total values of 5 numbers is 35. Next, we have to find the removed number :

[tex]let \: y = removed \: number[/tex]

[tex] \frac{35 - y}{4} = 6[/tex]

[tex]35 - y = 6 \times 4[/tex]

[tex]35 - y = 24[/tex]

[tex]35 - 24 = y[/tex]

[tex]y = 11[/tex]

Okay, let's slightly generalize this

Average of [tex]n[/tex] numbers is [tex]a[/tex]

and then [tex]r[/tex] numbers are removed, and you're asked to find the sum of these [tex]r[/tex] numbers.

Solution:

If average of [tex]n[/tex] numbers is [tex]a[/tex] then the sum of all these numbers is [tex]n\cdot a[/tex]

Now we remove [tex]r[/tex] numbers, so we're left with [tex](n-r)[/tex] numbers. and their. average will be [tex]{\text{sum of these } (n-r) \text{ numbers} \over (n-r)}[/tex] let's call this new average [tex] a^{\prime}[/tex]

For simplicity, say, sum of these [tex]r[/tex] numbers, which are removed is denoted by [tex]x[/tex] .

so the new average is [tex]\frac{\text{Sum of } n \text{ numbers} - x}{n-r}=a^{\prime}[/tex]

or, [tex] \frac{n\cdot a -x}{n-r}=a^{\prime}[/tex]

Simplify the equation, and solve for [tex]x[/tex] to get,

[tex] x= n\cdot a -a^{\prime}(n-r)=n(a-a^{\prime})+ra^{\prime}[/tex]

Hope you understand it :)

-8 + (-15)
Evaluate this expression ​

Answers

Answer:

-23

Step-by-step explanation:

-8+(-15) means that you are subtracting 15 from -8. So you end up with -8-15=-23.

what is the domain of f(x)=(1/4)^x

Answers

Answer:

B All real numbers

hope you wil understand

Answer:

[tex]\boxed{\sf B. \ All \ real \ numbers}[/tex]

Step-by-step explanation:

The domain is all possible values for x.

[tex]f(x)=(\frac{1}{4} )^x[/tex]

There are no restrictions on the value of x.

The domain is all real numbers.

Find the value of the expression: −mb −m^2 for m=3.48 and b=96.52

Answers

Answer:

The value of the expression when [tex]m = 3.48[/tex] and [tex]b = 96.52[/tex] is 323.779.

Step-by-step explanation:

Let be [tex]f(m, b) = m\cdot b - m^{2}[/tex], if [tex]m = 3.48[/tex] and [tex]b = 96.52[/tex], the value of the expression:

[tex]f(3.48,96.52) = (3.48)\cdot (96.52)-3.48^{2}[/tex]

[tex]f(3.48,96.52) = 323.779[/tex]

The value of the expression when [tex]m = 3.48[/tex] and [tex]b = 96.52[/tex] is 323.779.

Given the number of trials and the probability of success, determine the probability indicated: a. n = 15, p = 0.4, find P(4 successes) b. n = 12, p = 0.2, find P(2 failures) c. n = 20, p = 0.05, find P(at least 3 successes)

Answers

Answer:

A)0.126775 B)0.000004325376 C) 0.07548

Step-by-step explanation:

Given the following :

A.) a. n = 15, p = 0.4, find P(4 successes)

a = number of trials p=probability of success

P(4 successes) = P(x = 4)

USING:

nCx * p^x * (1-p)^(n-x)

15C4 * 0.4^4 * (1-0.4)^(15-4)

1365 * 0.0256 * 0.00362797056

= 0.126775

B)

b. n = 12, p = 0.2, find P(2 failures),

P(2 failures) = P(12 - 2) = p(10 success)

USING:

nCx * p^x * (1-p)^(n-x)

12C10 * 0.2^10 * (1-0.2)^(12-10)

66 * 0.0000001024 * 0.64

= 0.000004325376

C) n = 20, p = 0.05, find P(at least 3 successes)

P(X≥ 3) = p(3) + p(4) + p(5) +.... p(20)

To avoid complicated calculations, we can use the online binomial probability distribution calculator :

P(X≥ 3) = 0.07548

Question: The hypotenuse of a right triangle has a length of 14 units and a side that is 9 units long. Which equation can be used to find the length of the remaining side?

Answers

Answer:

The hypotenuse is the longest side in a triangle.

a^2=b^2+c^2.

14^2=9^2+c^2.

c^2=196-81.

c^2=115.

c=√115.

c=10.72~11cm

Other Questions
An "agnostic" is a person who does not believe there is enough evidence to know whether God exists. It is unknowable to that person.A. TrueB. False 67.77759 rounded to nearest meter Evaluate the following expression for x = 1 and y = -3.3yx+x-y Please answer and show steps. Will mark brainliest. Why princess Margaret couldn't marry Peter Townsend ? which stratgy makes the most sense when cost pressure are intesne and demands for local resosiveness is limited What is the distance between points A and B on the grid? (the grid is shown in the image). A. 3 units B. 4.5 units C. 5 units D. 5.5 units Juan works as a tutor for $12 an hour and as a waiter for $7 an hour. This month, he worked a combined total of 110 hours at his two jobs.Let t be the number of hours Juan worked as a tutor this month. Write an expression for the combined total dollar amount he earned this month. Explain how to overhand serve A boutique wants to make at least $127 profit from purses this week. The boutique earns $7 profit from each purse. How many purses must be sold? Given the following angles, what ray is the common side of ZCFD and ZDFE?DE0Ray FDRay FERay FC Which of the following statements is true of an encrypted file? * The file can be read only by a user with the decryption key The file can be read by the creator and system administrator The file can not be opened again The file can be opened but not read again pls help this is from khan academy For his persuasive speech, Tomas first identified all the objections to his position, and then he used evidence to argue against those objections. What organizational pattern is Tomas using for his speech I need help on answering this question If c o v (x comma space y )space equals space 1260, s subscript x squared equals 1600, and s subscript y squared equals 1225 , then the coefficient of determination is: hi guys!!! i have no more points, can someone nice guess all of these for me? :) 1.What happens to the ocean water before the precipitation part of the water cycle 2.During which stage of the water cycle does water from the ocean form clouds? 3.what is a runoff?? 4.Which statement about oceans is incorrect? A.Evaporation occurs when water is warmed by the sun. B.Most evaporation and precipitation occur over the ocean. C.97 percent of Earth's water is fresh water from the ocean. D.Water leaves the ocean by the process of evaporation 5.How does most ocean water return to the ocean in the water cycle tysm to u who answers :) erekes Manufacturing Corporation has prepared the following overhead budget for next month. Activity level 3,200 machine-hours Variable overhead costs: Supplies $ 16,640 Indirect labor 29,120 Fixed overhead costs: Supervision 15,400 Utilities 6,600 Depreciation 7,600 Total overhead cost $ 75,360 The company's variable overhead costs are driven by machine-hours. What would be the total budgeted overhead cost for next month if the activity level is 3,100 machine-hours rather than 3,200 machine-hours A ball is thrown upward from a height of 432 feet above the ground, with an initial velocity of 96 feet per second. From physics it is known that the velocity at time t is v (t )equals 96 minus 32 t feet per second. a) Find s(t), the function giving the height of the ball at time t. b) How long will the ball take to reach the ground? c) How high will the ball go? Use the grouping method to factor x3 + x2 + 2x + 2.